LSAT and Law School Admissions Forum

Get expert LSAT preparation and law school admissions advice from PowerScore Test Preparation.

 Administrator
PowerScore Staff
  • PowerScore Staff
  • Posts: 8916
  • Joined: Feb 02, 2011
|
#41623
Complete Question Explanation
(The complete setup for this game can be found here: lsat/viewtopic.php?t=15887)

The correct answer choice is (A)

If J is earlier than G, then K must be the first, because either K or G must be first. And, since G must precede the HF rotating block, J must perform second, and G—third. We can use a linear setup to represent the fixed placement of these variables:
PT65_D11 LG Explanations_game_#1_#2_diagram 1.png
The only variables whose position is relatively unknown are H and F. Because we are solving a Could Be True question, we should expect that the correct answer choice will exploit this uncertainty—a prephrase that immediately eliminates all answer choices except for answer choices (A) and (C).

Answer choice (A): This is the correct answer choice, as F could perform either fourth or fifth.

Answer choice (B) cannot be true, because G must perform third, after K and J.

Answer choice (C) cannot be true, because H must perform either fourth or fifth.

Answer choice (D) cannot be true, because J must perform second.

Answer choice (E) cannot be true, because K must perform first.
You do not have the required permissions to view the files attached to this post.

Get the most out of your LSAT Prep Plus subscription.

Analyze and track your performance with our Testing and Analytics Package.